Développement asymptotique — Les-mathematiques.net The most powerful custom community solution in the world

Développement asymptotique

Bonjour
Dans un exercice d'oral ENTPE MP, on définit la suite (In) par : pour tout entier naturel n, In = intégrale de 0 à 1 de x^n/(1+x) dx.
Il est demandé de trouver a et b réels tels que In = a/n + b/n² + o(1/n²).

Ma démarche : on prouve la décroissance de (In) et le fait que In+ In+1 = 1/(n+1) conduisent sans peine à In équivaut à 1/2n au voisinage de l'infini, donc In = 1/2n + o(1/n).

Comment obtenir le terme en 1/n² ? J'ai posé un = In - 1/2n mais là je patine, j'arrive seulement à un encadrement grossier :
-1/2n(n+1) <=un<=0.

Merci pour votre réponse et belle journée à tous.
gauss.

Réponses

  • Pour plus de lisibilité $I_n=\int_{0}^{1} \frac{x^n}{1+x} dx$
  • Bonjour,

    Deux intégrations par parties (et une majoration d’intégrale) devraient permettre de conclure.
  • Bonjour

    je propose une solution basée sur les séries numériques :

    l'intégrale I(n) s'écrit : $\int_0^1\frac{x^n}{1+x}dx = \int_0^1(x^n - x^{n+1} + x^{n+2} - x^{n+3} +........)dx$

    soit encore sous forme de développement rationnel :$\frac{1}{n+1} - \frac{1}{n+2}+\frac{1}{n+3} - \frac{1}{n+3} +........$

    ou encore en mettant 1/(n+1) en facteur : $\frac{1}{n+1}[1 - \frac{n+1}{n+2} + \frac{n+1}{n+3} - \frac{n+1}{n+4} + ........]$

    et en décomposant chaque fraction il vient 2 séries à l'intérieur du crochet :

    $I(n) = \frac{1}{n+1}[1 - 1 + 1 - 1 + .....+ \frac{1}{n+2} - \frac{2}{n+3} + \frac{3}{n+4} - \frac{4}{n+5} +.........]$

    et compte tenu du résultat 1/2 de la première série et en prenant les équivalents asymptotiques des fractions de la seconde série il vient :

    $I(n)$ équivalent à $\frac{1}{n}[\frac{1}{2} + \frac{1}{n} - \frac{2}{n} + \frac{3}{n} - \frac{5}{n} + .........]$ soit encore $I(n)$ équivalent à

    $$\frac{1}{2n} + \frac{1}{4n^2}$$

    en effet on a reconnu la série numérique $\frac{1}{4} = 1 - 2 + 3 - 4 +......$ qui représente le premier nombre de Bernoulli

    Cordialement
  • J'ai trouvé $I_n = \dfrac{1}{2n} + \dfrac{5}{2n^2} + o_{\infty}\bigg( \dfrac{1}{n^2} \bigg)$ sans aucune série.
  • Bonsoir
    Il suffit de réaliser deux intégrations par parties:
    Dans un premier temps, si on pose $v':=x^{n}$ et $u:=\frac{1}{1+x}$
    Puis on pose $f':=x^{n+1}$ et $g:=\frac{1}{(x+1)^2}$.
    on trouve que $I_{n}=\frac{1}{2n}-\frac{1}{4n^2}+J_{n}$ où $J_{n}=o(\frac{1}{n^2})$
    HT, comment obtiens-tu le 5 au numérateur?
  • En faisant exactement cette IPP, j'ai abouti à $I_n = \dfrac{1}{2(n+1)} + \dfrac{1}{2(n+1)(n+2)}\bigg(1+4\displaystyle \int_0^1 \dfrac{x^{n+2}}{(1+x)^3}dx\bigg)$.

    En supposant que je n'ai pas fait d'erreur de calcul, j'ai juste majoré l'intégrale par $1$ comme un bourrin.

    EDIT : je crois que je sais pourquoi ça ne marche pas. Mais je n'ai pas encore trouvé comment faire un truc qui marche, alors.
  • Bonsoir,

    Puisque la méthode a été détaillée, je propose mes calculs. Je trouve quant à moi un signe moins:

    $ I_n = \displaystyle \int_0^1 \frac{x^n}{1+x}dx = [ \frac{x^{n+1}} {(n + 1)(1 + x)} ]_0^1 + \frac{1}{n + 1} \int_0^1 \frac{x^{n + 1}}{(1 + x)^2} dx $, et:

    $ \displaystyle \int_0^1 \frac{x^{n + 1}}{(1 + x)^2} dx = [ \frac{x^{n+2}}{(n + 2)(1 + x)^2} ]_0^1 + 2 \int_0^1 \frac{x^{n+2}}{(n + 2)(1+x)^3} dx$, donc:

    $I_n = \frac{1}{2(n + 1)} + \frac{1}{n + 1} ( \frac{1}{4(n + 2)} + o(\frac{1}{n}))$.

    Or $\frac{1}{2(n + 1)} = \frac{1}{2n} - \frac{1}{2n^2} + o(\frac{1}{n^2})$ et $\frac{1}{n + 1} \times \frac{1}{4(n + 2)} = \frac{1}{4n^2} + o(\frac{1}{n^2})$

    Donc $I_n = \frac{1}{2n} - \frac{1}{4n^2} + o(\frac{1}{n^2})$.

    Ceci est cohérent avec la formule $I_n + I_{n - 1} = \frac{1}{n}$ qui implique $0 < 2I_n \leq I_n + I_{n - 1} = \frac{1}{n}$ donc $0 < I_n \leq \frac{1}{2n}$.
  • J'avais bien fait une erreur de calcul (enfin, d'étourderie, pour changer : oubli de mettre le $2$ au carré au dénominateur dans le crochet...).

    Le truc qui m'intrigue le plus dans ce que tu as écrit, et ce n'est pas une blague, c'est le $\dfrac{1}{2(n+1)} = \dfrac{1}{2n} - \dfrac{1}{2n^2} + o \Big( \dfrac{1}{n^2} \Big)$. D'où ça sort ???
  • $ \frac{1}{2(n+1)} = \frac{1}{2n} ( 1 + \frac{1}{n} )^{-1} = \frac{1}{2n}(1 - \frac{1}{n} + o(\frac{1}{n})) $
  • La première égalité, je suis d'accord, mais la deuxième ???
  • Si on veut c’est le développement limité à l’ordre 1 en 0 de $ x \mapsto (1 + x)^a$ avec ici $ a = -1$.

    Sinon directement via $\frac{1}{1+x} = 1 - x + x^2 - ...$ si $ \lvert x \rvert < 1$ ce qui fournit en fait un développement limité en 0.
  • On peut aller plus loin : $I_n = \dfrac{1}{2n} - \dfrac{1}{4n^2}+ \dfrac{1}{8n^4} + o\left(\dfrac{1}{n^4}\right)$.

    Il existe une formule générale qui fait intervenir les puissances de $2n+1$ : $I_n=\displaystyle\sum_{k=0}^p\dfrac{(-1)^ke_{2k}}{(2n+1)^{2k+1}}+o\left(\dfrac1{(2n+1)^{2p+1}}\right)$
    où les $e_{2k}$ sont les nombres d'Euler qui apparaissent dans le développement en série entière $\dfrac1{\cos x}=\displaystyle\sum_{k=0}^{+\infty}e_{2k}\dfrac{x^{2k}}{(2k)!}$.
  • Je m'étais fait la réflexion qu'il suffit de continuer les IPP pour avoir un équivalent aussi précis que l'on veut. Mais je ne m'attendais pas à une formule comme ça.
  • Je trouve comme Jandri $I_n = \dfrac{1}{2n} - \dfrac{1}{4n^2}+ \dfrac{1}{8n^4} + o\left(\dfrac{1}{n^4}\right)$.
    Le développement asymptotique des $\frac 1{(n+1)(n+2)...(n+p)}$, dans l'échelle des $\frac 1{n^q}$, quand $n \rightarrow +\infty$, est donné par le développement en série entière des $\frac 1{(1-x)(1-2x)...(1-px)}$, qui a pour coefficients les nombres de Stirling de seconde espèce.
  • Homo Topi, tu as raison, la suite des IPP fournit, disons plutôt, un développement asymptotique de $I_n$ aussi précis que l'on veut.
    En effet, soit $\displaystyle R_{n,p}=\int_{0}^{1}\frac{x^{n+p}}{(1+x)^{p+1}}dx=o(1)$ quand $n\rightarrow
    +\infty $. Alors l'IPP donne : $R_{n,p}=\frac{1}{2^{p}(n+p+1)}+\frac{p+1}{n+p+1}R_{n,p+1}$,
    D'où : $\displaystyle I_{n}=\int_{0}^{1}\frac{x^{n}}{1+x}dx=R_{n,0}=\overset{p}{\underset{k=1}{\sum }}\frac{(k-1)!}{%
    2^{k}(n+1)(n+2)...(n+k)}+\frac{p!}{(n+1)(n+2)...(n+p)}R_{n,p}$
    $~~~~~~~~~~~~~~~~~~~~~~~~\displaystyle =\overset{p}{\underset{k=1}{\sum }}\frac{(k-1)!}{2^{k}(n+1)(n+2)...(n+k)}+o(\frac1{n^p})$.
    Mais ce développement n'apparaît pas spontanément comme combinaison linéaire des $\frac1{n^q}$, et c'est bien là l'ennui.
    Bah, ça nous fait une occasion de calculer.
    Bonne journée, enfin estivale.
    Fr. Ch.
  • Oui, je pensais à un développement asymptotique aussi précis que l'on veut, je ne sais pas pourquoi j'ai dit "équivalent".
  • Le développement asymptotique de $I_n$ calculé par Chaurien ne fait pas apparaitre les $\dfrac1{n^k}$ mais c'est mieux qu'un développement asymptotique puisque c'est une série convergente : $I_n=\displaystyle\sum_{k=1}^{+\infty}\frac{(k-1)!}{2^{k}(n+1)(n+2)...(n+k)}$ car $0\leq \dfrac{p!}{(n+1)(n+2)...(n+p)}R_{n,p}\leq\dfrac1{p+1}$.
  • Merci à etanche pour le lien avec la solution de Start wearing purple : même si le résultat est écrit comme la somme d'une série divergente cela donne bien un un développement asymptotique de $I_n$ aussi précis que l'on veut.

    Le point de départ est le changement de variable $x=e^{-t/n}$ qui conduit à $\displaystyle I_{n}=\int_{0}^{1}\dfrac{x^{n}}{1+x}dx=\dfrac1{2n}\int_0^{+\infty}e^{-t}(1-\tanh\dfrac t{2n})dt$.

    La formule de Taylor avec reste intégral donne $\tanh x=\displaystyle\sum_{k=0}^{p-1}\dfrac{a_k x^{2k+1}}{(2k+1)!}+R_p(x)$ avec $a_k=\tanh^{(2k+1)}(0)$ et $|R_p(x)|\leq M_{2p}\dfrac{ x^{2p}}{(2p)!}$ puisque la dérivée $2p$-ème de la fonction $\tanh$ est un polynôme en $\tanh x$ et que $|\tanh x|\leq1$.

    On en déduit en posant $x=\dfrac t{2n}$ et en intégrant : $I_n=\dfrac1{2n}-\displaystyle\sum_{k=0}^{p-1}\dfrac{a_k}{(2n)^{2k+2}}+o\left(\dfrac1{(2n)^{2p}}\right)$.

    Les nombres $a_k$ sont connus puisque $a_k=(-1)^kT_{2k+1}$ où les $T_{2k+1}$ sont les nombres tangents (c'est la suite A182 de l'OEIS) : $\tan x=\displaystyle\sum_{k=0}^{+\infty}\dfrac{T_{2k+1}}{(2k+1)!}x^{2k+1}$ pour $|x|<\dfrac{\pi}2$. On les calcule par la récurrence $\displaystyle\sum_{k=0}^n(-1)^k{2n+1\choose 2k+1}T_{2k+1}=1$.


    Si on considère maintenant la suite $J_n=\displaystyle \int_{0}^{1}\dfrac{x^{n}}{1+x^2}dx$, le même changement de variable conduit à $J_n=\dfrac1{2n}\displaystyle\int_0^{+\infty}\dfrac{e^{-t}}{\cosh\dfrac t{2n}}dt$.
    La formule de Taylor appliquée à la fonction $x\mapsto\dfrac1{\cosh x}$ conduit au développement asymptotique :
    $J_n=\displaystyle\sum_{k=0}^p\dfrac{b_k}{(2n)^{2k+1}}+o\left(\dfrac1{(2n)^{2p+1}}\right)$ avec $b_k=\left(\dfrac1\cosh\right)^{(2k)}(0)=(-1)^k e_{2k}$ où les $e_{2k}$ sont les nombres sécants d'Euler (c'est la suite A364 de l'OEIS) : $\dfrac1{\cos x}=\displaystyle\sum_{k=0}^{+\infty}\dfrac{e_{2k}}{(2k)!}x^{2k}$ pour $|x|<\dfrac{\pi}2$. On les calcule avec $e_0=1$ et la récurrence $\displaystyle\sum_{k=0}^n(-1)^k{2n\choose 2k}e_{2k}=0$ si $n\geq1$.
  • Potentiellement bête, mais je n'arrive pas à montrer que $\int_0^1 \frac{x^{n+2}}{(1+x)^3}dx$ tend vers $0$ sans faire une IPP de plus (et du coup montrer que c'est un $O(\frac{1}{n})$, ce qui n'est de toute évidence pas nécessaire) à cause du défaut de convergence uniforme en $1$...

    Edit : on est sur un compact, je suis rouillé, oubliez-moi svp... B-)
  • @jandri de rien. C’est toujours très agréable de lire tes posts de mathématiques.
    @Victor2N
    $$0 \leq \int_0^1 \frac{x^{n+2}}{(1+x)^3}dx \leq \int_{0}^{1} x^{n+2} dx = \frac{1}{n+3}.

    $$ Donc l’intégrale tend vers 0.
Connectez-vous ou Inscrivez-vous pour répondre.
Success message!